Skip to main content

Questions tagged [quantum-field-theory]

Quantum Field Theory (QFT) is the theoretical framework describing the quantisation of classical fields which allows a Lorentz-invariant formulation of quantum mechanics. QFT is used both in high energy physics as well as condensed matter physics and closely related to statistical field theory. Use this tag for many-body quantum-mechanical problems and the theory of particle physics. Don’t combine with the [quantum-mechanics] tag.

Filter by
Sorted by
Tagged with
-4 votes
0 answers
42 views

Is it possible for entanglement to be a dynamic field, suggesting that particles become entangled through their interaction with this field, as opposed to viewing entanglement as a direct, passive ...
QuantumOptics's user avatar
0 votes
0 answers
28 views

I am trying to do the canonical quantization of an abelian vector field $A_\mu(x)$ in the $R_\xi$ gauge. So the gauge-fixing Lagrangian is given by $$ \mathscr{L}_{gf} = -\frac{1}{2\xi} (\partial_\mu ...
physmath17's user avatar
-4 votes
0 answers
53 views

After reading Noether's theoram and many other examples like objects tendency to follow its geodesics in spacetime makes conserved quantities like momentum etc meaningful so , are energy and momentum ...
Gufran's user avatar
  • 1
4 votes
2 answers
282 views

In an attempt to understand the Reeh-Schlieder theorem, I am currently studying this paper, in which Witten provides a discussion using the formalism of quantum field theory. It is quite ...
flippiefanus's user avatar
  • 17.7k
5 votes
4 answers
781 views

I struggle to understand how theories that are based on renormalization can be considered mathematically rigorous. I understand how renormalization works for non-abelian theories, through loop ...
Timur Obolenskiy's user avatar
1 vote
0 answers
57 views

Consider Compton scattering $$p_1 + p_2 \to p_3 + p_4$$ in the laboratory frame. According to Quantum Field Theory and the Standard Model by Schwartz, the relation between the differential cross ...
pll04's user avatar
  • 486
2 votes
1 answer
89 views

Let's say we describe an unstable particle using perturbation theory. Then we have a non-zero decay width, which we say $\Gamma$. Now, if we define mass to be the pole of the propagator, we get $$ \mu^...
Tanmoy Pati's user avatar
1 vote
1 answer
92 views

I am working on a problem about Cutkosky’s cutting rule in Matt D. Schwartz’s Quantum Field Theory and the Standard Model. The problem asks us to show that the imaginary part of the amplitude is given ...
Jaeok Yi's user avatar
  • 354
3 votes
1 answer
128 views

It seems this has been discussed here previously (e.g this and this post), but I still feel uneasy. Essentially, I think it all boils down to the popular interpretation of the scalar field operator in ...
Lourenco Entrudo's user avatar
17 votes
8 answers
2k views

UPDATE for Mods: Currently travelling; aiming for edited version over weekend 30 Nov. After reading a number of posts here in the last few years I'm left wondering why most questions about the ...
iSeeker's user avatar
  • 1,370
2 votes
1 answer
102 views

I am trying to practice the techniques given in the book "Feynman Integrals: A Comprehensive for Students and Researchers," by Stefan Weinzierl (preprint). I am getting stuck on one point ...
DiracComb16796's user avatar
0 votes
0 answers
102 views

I'm studying the renormalization of scalar quantum field theories ($\lambda\phi^4$ in particular). I'm considering renormalization by counterterms with the old non-Wilsonian interpretation of ...
HomoVafer's user avatar
  • 864
5 votes
0 answers
152 views

The Higgs potential is written as $V(\phi) = -\mu^2 |\phi|^2 + \lambda^2 |\phi|^4$, where $|\phi|^2 = \phi^\dagger \phi $ and $ \phi $ is a complex scalar doublet. My question is: why do we not ...
Martin's user avatar
  • 51
3 votes
1 answer
205 views

In Fetter and Walecka's Quantum Theory of Many-Particle Systems, after a discussion of Dyson's equation for the single particle Green's function in spacetime ($x$) and momentum/frequency ($k$; for ...
frobenius's user avatar
  • 169
1 vote
0 answers
93 views

On Chapter 7 of Fetter & Walecka, the authors prove Dyson formula for the (imaginary time) propagator $U(t,t_{0}) = e^{H_{0}t_{0}}e^{-H(t_{0}-t)}e^{-H_{0}t}$, where I am ommitting the $\hbar$'s. ...
InMathweTrust's user avatar
0 votes
0 answers
58 views

In Peskin & Schroeder (Eq.~(19.88)), the PCAC matrix element is written as $\langle 0| j^{\mu 5 a}(x) | \pi^b(p) \rangle = -\, i\, p^\mu f_\pi\, \delta^{ab}\, e^{-ip\cdot x}.$ Q1. Where does this ...
Namichi's user avatar
  • 23
2 votes
0 answers
77 views

According to Weinberg' QFT vol I, (2.4.8), the generator of angular momentum $J^{\rho\sigma}$ transform as $$U(\Lambda, a) J^{\rho \sigma} U^{-1}(\Lambda,a) =\Lambda_\mu^{\,\,\rho} \Lambda_\nu^{\,\,\...
orange's user avatar
  • 63
6 votes
1 answer
267 views

In Quantum Theory of Many Particle Systems by Fetter and Walecka, they perturbatively expand the fermionic interacting single particle Green's function $iG_{\alpha\beta}(x, y)$ (where $\alpha$, $\beta$...
frobenius's user avatar
  • 169
3 votes
1 answer
307 views

In a QFT with Euclidean signature, the correlation functions can only be well-defined in a time-ordered manner (This is Claim 1 on Page 2 of Simmons-Duffin's lecture note). For example, a scalar 2pt ...
Hezaraki 's user avatar
3 votes
2 answers
72 views

Consider a 2-to-2 particles scattering process (treated in second quantization) of particles with the following $4$-momenta: $$p_1 + p_2 \rightarrow p_3 + p_4.$$ The probability measure associated ...
pll04's user avatar
  • 486
1 vote
1 answer
92 views

I'm trying to understand a conceptual point in many-body quantum mechanics and quantum field theory. Starting point: Consider a classical Schrödinger field $\psi(\mathbf r, t)$ with interactions, ...
Michiel's user avatar
  • 310
13 votes
4 answers
2k views

I am curious as to why we find Dirac spinors in QED but when working in CED we can formulate systems using complex scalar fields. For instance, the typical Lagrangian density for a charged particle ...
Lagrangiano's user avatar
  • 2,646
1 vote
0 answers
99 views

Dear members of the condensed matter community, There are two things in the non-relativistic quantum field theory of solids that I cannot quite reconcile: (1) We start from the full crystal ...
Michiel's user avatar
  • 310
0 votes
1 answer
128 views

In Why particles only make sense in flat spacetime? user Chiral Anomaly says the Reeh-Schlieder thereom implies that the vacuum state (the lowest-energy state of the global Hamiltonian $H$) cannot be ...
Nairit Sahoo's user avatar
3 votes
1 answer
129 views

$\Bbb {CP}^1$ the Riemann sphere appears in quantum physics as the Bloch sphere, and is the natural home for two-dimensional phenomena such as spin. It is sometimes said that bundle theory is relevant ...
FShrike's user avatar
  • 272
2 votes
0 answers
41 views

Page 370 (start of section 11.4) of Peskin and Schroeder claims that the VEV of a scalar field in the presence of an external source, $$\phi_\text{cl} \equiv \langle 0_J|\phi(x)|0_J\rangle,\tag{11.46}$...
Aryan MP's user avatar
  • 147
2 votes
1 answer
120 views

I am a math student taking a course in fundamental interactions and I have two questions about the interpretation of the spontaneous symmetry breaking (SSB). For contest we started with a classical ...
Trinity-Slifer 's user avatar
-2 votes
0 answers
69 views

I'm trying to solidify my understanding of the path integral formalism in Quantum Field Theory, and I've run into a conceptual paradox regarding the definition of initial states. I would be grateful ...
J.Doe's user avatar
  • 129
0 votes
0 answers
28 views

I am studying the Effective Field Theory for extended objects proposed by Goldberger and Rothstein. This theory studies a binary system in inspiral phase. By means of the virial theorem we get a ...
julian's user avatar
  • 37
3 votes
1 answer
130 views

I've heard about neutrino oscillation, where can neutrino can change between each flavour. I've also heard that the neutrino flavours are not guaranteed to have the same mass. If this is the case how ...
blademan9999's user avatar
  • 3,645
6 votes
2 answers
592 views

In the book What is a Quantum Field Theory by Michel Talagrand, he introduces (in §10.15) a "massive Weyl spinor."* In physics, common wisdom says there is no such thing. A quick internet ...
WillG's user avatar
  • 3,908
3 votes
1 answer
185 views

Given a Lagrangian $\mathcal{L} = \mathcal{L}_0 + \mathcal{L}_I$, we can construct the Feynman diagrams for some process by writing out the Taylor series for our interaction term and judiciously ...
wlancer's user avatar
  • 404
2 votes
0 answers
51 views

I am doing calculations with a resonance with the invariant mass $W$, which can decay to $a + b$ with masses $m_a$ and $m_b$. Since I treat all particles as scalar, I calculate with $\ell=0$. The ...
Xorker's user avatar
  • 21
6 votes
1 answer
377 views

I’m a physics major working on magnon–phonon mediated superconductivity. I have a solid foundation in many-body theory and field theory, but I’m looking for good reading material or notes on ...
1 vote
0 answers
56 views

I am performing numerical calculations using quantum many-body theory, and I would like to fix the particle number, either for fermions or bosons. The particle number can be expressed in terms of the ...
Black Monolith's user avatar
1 vote
1 answer
144 views

We can show that for the Dirac field, the anti-commutator between the field and its adjoint vanishes for space-like separated points. However, for causality we need to show that the commutator instead ...
Principia Mathematica's user avatar
1 vote
2 answers
177 views

Reading Feynman rules from a Lagrangian is a quite standard procedure. However I have seen papers (for example Appendix A of arXiv:2412.14858) where this is done from the Equations of Motion instead ...
Colin's user avatar
  • 21
0 votes
2 answers
884 views

Photon in QFT is e.g. a coupling between two electrons using below Feynman diagram - requires both emitter and absorber, they are switched in perspective of CPT symmetry. So can photon be emitted if ...
Jarek Duda's user avatar
  • 1,196
4 votes
1 answer
272 views

In quantum field theory, the path integral for a bosonic field has a very intuitive interpretation as a "sum over all possible field configurations." To make this concrete, let's consider a ...
particle-not good at english's user avatar
5 votes
1 answer
195 views

recently I was revisiting the path integral formalism to approach condensed matter systems, and there is a question I've always had which is not yet clear to me. In physics, there are many ways to ...
Eduardo Sánchez's user avatar
1 vote
0 answers
81 views

In Polchinski's book, it states that the corresponding operators of $|1\rangle, |-1\rangle$ are $\delta(\beta),\delta(\gamma)$, and suggests that it can be shown by path integral. I'm a little ...
Sirin's user avatar
  • 19
3 votes
1 answer
325 views

I read the Wiki page about the reweighting procedure as a way to use Monte Carlo methods with the sign problem, but I'd like to know more about how this could be implemented in the Metropolis ...
2 votes
1 answer
191 views

I'm following notations/conventions of Srednicki, chapter 3. My question relates to some conserved currents and charges of the free real scalar field, that arise in addition to the translation and ...
Oliver Gray's user avatar
2 votes
0 answers
127 views

There are plenty of discussions on this site about why particles correspond to representations (reps) of the Poincaré group $\mathcal P$. My question (perhaps quite naive) is why we consider $\mathcal ...
WillG's user avatar
  • 3,908
4 votes
1 answer
142 views

I understand that the commutator of field operators in a free field theory is always a $c$-number. Now I want to see whether a generalization to interacting fields is also possible? In the interacting ...
Frank's user avatar
  • 51
4 votes
0 answers
59 views

In the functional-renormalization-group (FRG) formulation of quantum gravity, the effective average action Γₖ evolves with the RG scale k according to the Wetterich equation $$\partial_k \Gamma_k = \...
nonymous's user avatar
  • 187
1 vote
1 answer
118 views

The real-valued Higgs field potential is usually described as a double well, where below a certain temperature the ball has to roll in one direction and go to a local minimum. There are two local ...
sashoalm's user avatar
  • 771
2 votes
0 answers
75 views

In the context of the pertubative expansion of Green's functions in Quantum Field Theory, it is a pretty standard trick to assume the free and interacting Hamiltonian are conected via adiabatic ...
Cement's user avatar
  • 89
0 votes
0 answers
51 views

Some context In these notes on AdS/CFT by Jared Kaplan (section 8.1) on pg. 70-71, he tries to derive anomalous dimensions from basic quantum mechanics and the idea is simple: We are doing ...
Nairit Sahoo's user avatar
3 votes
1 answer
193 views

This question concerns the definition of dimensional regularization in quantum field theory, specifically as presented in this Wilson paper (see free version here). This operation must fulfill three ...
Gaussian97's user avatar

1
2 3 4 5
291